Is the set of paths between any two points moving only in units on the plane countable or uncountable?The set...

Are there any other Chaos-worshipping races?

School performs periodic password audits. Is my password compromised?

Second-rate spelling

Reason why dimensional travelling would be restricted

What could trigger powerful quakes on icy world?

lead or lag function to get several values, not just the nth

Levi-Civita symbol: 3D matrix

What's the values for the Freq_Interval column in MSDB.dbo.SysSchedules when Freq_Type is weekly and more than one day is selected in the schedule?

Where is the line between being obedient and getting bullied by a boss?

In Adventurer's League, is it possible to keep the Ring of Winter if you manage to acquire it in the Tomb of Annihilation adventure?

Real life puzzle: Unknown alphabet or shorthand

Can I become debt free or should I file for bankruptcy? How do I manage my debt and finances?

Skis versus snow shoes - when to choose which for travelling the backcountry?

Plagiarism of code by other PhD student

The need of reserving one's ability in job interviews

I can't die. Who am I?

What type of investment is best suited for a 1-year investment on a down payment?

How can atoms be electrically neutral when there is a difference in the positions of the charges?

How to evaluate the limit where something is raised to a power of x?

Misplaced tyre lever - alternatives?

Difference between 'stomach' and 'uterus'

Can throughput exceed the bandwidth of a network

Borrowing Characters

What are the issues with an additional (limited) concentration slot instead of Bladesong?



Is the set of paths between any two points moving only in units on the plane countable or uncountable?


The set of points that are equdistiant from two circlesproof of the fact that the set of points equidistant from sides of an angle form a bisector of the angleFinding a point between two points given the points and a distanceWhat is the smaller angle made by the two radial paths?Compute the angle between a line and a plane if the line forms the angles of 45 degrees and 60 degrees with two perpendicular lines lying in the planeProve any line passes through at least two pointsHow many points are possible in the plane that are the same distance from all three points.Existence of a point between two points in Hilbert geometryPoints moving towards the nearest point, where will they meet?Shortest distance between two points , when you can't cross sphere in the middle













13












$begingroup$


Consider 2 arbitrary, fixed points A and B on the plane. Suppose you can move from point A in unit distance at any angle to another point and from this point you can again travel a unit distance at any angle to another point and so on. Ultimately your goal is to travel from point A to point B along a path of unit-distance line segments without repeating a point during your journey. Is the set of such paths countable or uncountable?



I believe it is uncountable and here is my thought process, but I'm not sure of my logic. Consider the line between the two points equidistant from them; call this line L. A path of only unit distances can be made between A and any point, P, which lies on L without crossing L(I don't know how to prove this statement but it seems true). This path can be mirrored on the other side of L to connect B to P. Thus for any point, P, which lies on L a path can be made from A to B crossing through P halfway through the path. Since the points on L are uncountable the set of paths between A and B are also uncountable.










share|cite|improve this question











$endgroup$

















    13












    $begingroup$


    Consider 2 arbitrary, fixed points A and B on the plane. Suppose you can move from point A in unit distance at any angle to another point and from this point you can again travel a unit distance at any angle to another point and so on. Ultimately your goal is to travel from point A to point B along a path of unit-distance line segments without repeating a point during your journey. Is the set of such paths countable or uncountable?



    I believe it is uncountable and here is my thought process, but I'm not sure of my logic. Consider the line between the two points equidistant from them; call this line L. A path of only unit distances can be made between A and any point, P, which lies on L without crossing L(I don't know how to prove this statement but it seems true). This path can be mirrored on the other side of L to connect B to P. Thus for any point, P, which lies on L a path can be made from A to B crossing through P halfway through the path. Since the points on L are uncountable the set of paths between A and B are also uncountable.










    share|cite|improve this question











    $endgroup$















      13












      13








      13


      2



      $begingroup$


      Consider 2 arbitrary, fixed points A and B on the plane. Suppose you can move from point A in unit distance at any angle to another point and from this point you can again travel a unit distance at any angle to another point and so on. Ultimately your goal is to travel from point A to point B along a path of unit-distance line segments without repeating a point during your journey. Is the set of such paths countable or uncountable?



      I believe it is uncountable and here is my thought process, but I'm not sure of my logic. Consider the line between the two points equidistant from them; call this line L. A path of only unit distances can be made between A and any point, P, which lies on L without crossing L(I don't know how to prove this statement but it seems true). This path can be mirrored on the other side of L to connect B to P. Thus for any point, P, which lies on L a path can be made from A to B crossing through P halfway through the path. Since the points on L are uncountable the set of paths between A and B are also uncountable.










      share|cite|improve this question











      $endgroup$




      Consider 2 arbitrary, fixed points A and B on the plane. Suppose you can move from point A in unit distance at any angle to another point and from this point you can again travel a unit distance at any angle to another point and so on. Ultimately your goal is to travel from point A to point B along a path of unit-distance line segments without repeating a point during your journey. Is the set of such paths countable or uncountable?



      I believe it is uncountable and here is my thought process, but I'm not sure of my logic. Consider the line between the two points equidistant from them; call this line L. A path of only unit distances can be made between A and any point, P, which lies on L without crossing L(I don't know how to prove this statement but it seems true). This path can be mirrored on the other side of L to connect B to P. Thus for any point, P, which lies on L a path can be made from A to B crossing through P halfway through the path. Since the points on L are uncountable the set of paths between A and B are also uncountable.







      geometry elementary-set-theory






      share|cite|improve this question















      share|cite|improve this question













      share|cite|improve this question




      share|cite|improve this question








      edited yesterday









      Andrés E. Caicedo

      65.6k8159250




      65.6k8159250










      asked yesterday









      Knight98Knight98

      955




      955






















          7 Answers
          7






          active

          oldest

          votes


















          10












          $begingroup$

          Yes, this looks convincing. For the missing step, go directly from A towards P in unit steps until the distance left is less than 2. Then use the remaining distance as the base of an isosceles triangle with unit legs, which you make point away from L.






          share|cite|improve this answer









          $endgroup$





















            5












            $begingroup$

            Go one unit from $a$ at an angle of $t$ to $c$.

            Go in unit steps along ca until one is less than a unit away from $b$ to a point $p$.

            If $p neq b$, then draw a triangle with base $pb$ and sides of unit length adding the sides as the final steps.



            As for each $t$ in $[0,2pi)$, I've constructed a different accepted zigzaging from $a$ to $b$, there are uncountably many ways of so staggering from $a$ to $b$.






            share|cite|improve this answer











            $endgroup$





















              2












              $begingroup$

              Let $C$ be on the line through $B$ that is perpendicular to the segment $AB$ with the distance $BC$ equal to $1/2.$ Take any half-line $L$, not through $B$, that originates at $A$ and intersects the segment $BC.$ For some $nin Bbb N$ there is a path along $L,$ starting at $A,$ determined by $n$ points $A=A_1,...,A_n$ where $A_j,A_{j+1}$ are distance $1$ apart for each $j<n,$ and such that the distance from $A_n$ to $B$ is less than $1.$



              Let the point $D$ be such that $A_nD=BD=1$ nd $Dnot in {A_1,...,A_n}.$ Then the path determined by ${A_1,...,A_n}cup {D}$ is a path of the desired type.



              The cardinal of the set all such $L$ is $2^{aleph_0}$ so there at least this many paths of the desired type, joining pairs of points .



              And each path is determined by a function from some ${1,2,...,m}subset Bbb N$ into $Bbb R^2.$ The set of all such functions has cardinal $2^{aleph_0}$ so there are at most $2^{aleph_0}$ paths of the desired type.






              share|cite|improve this answer











              $endgroup$





















                1












                $begingroup$

                Suppose first that the distance $AB$ between $A$ and $B$ is strictly between $2$ and $3$. Then for every point $C$ such that $AC=1$ and $BC<2$, there is a second point $D$ such that $CD=1=BC$ (draw an isosceles triangle). The set of such points $C$ forms an open arc on the unit circle centered at $A$, and so there are uncountably many such $C$, hence uncountably many such "unit paths".



                For general points $A$ and $B$, just find one unit path between $A$ and $B$ that contains two intermediate points $A'$ and $B'$ with $2<A'B'<3$ (for example, points on a giant circle eventually connecting $A$ to $B$), and then vary the path from $A'$ to $B'$ in the way described above.



                This proof even shows that one can get uncountably many unit paths with the same number of unit-steps. A slight sharpening shows that for any integer $k>AB$, there are uncountably many unit paths from $A$ to $B$ with exactly $k$ unit-steps.






                share|cite|improve this answer









                $endgroup$





















                  0












                  $begingroup$

                  I believe it is uncountable also, and here is my thought process:



                  For each direction vector $vinBbb R^2=T_aBbb R^2$, take a curve $alpha_v$ from $a$ to $b$ with $alpha_v'(0)=v$.



                  Then if $vneq w$, we have $alpha_vneqalpha_w$.



                  But clearly there are uncountably many direction vectors in $Bbb R^2=T_aBbb R^2$.






                  share|cite|improve this answer









                  $endgroup$













                  • $begingroup$
                    It seems OP wants movements made of straight line segments all of length $1,$ to go from fixed $A$ to fixed $b$ points. [rather than curves as you use]
                    $endgroup$
                    – coffeemath
                    yesterday










                  • $begingroup$
                    I did notice that. But I thought, well, straight lines have derivatives. @coffeemath
                    $endgroup$
                    – Chris Custer
                    yesterday










                  • $begingroup$
                    Cris-- Then to finish one has to show that after the first initial unit straight line step from $A,$ it is possible to continue such steps and somehow arrive at $B.$
                    $endgroup$
                    – coffeemath
                    23 hours ago










                  • $begingroup$
                    @coffeemath yes. Well there certainly appear to be some details missing. But this seems fairly reasonable. Thanks.
                    $endgroup$
                    – Chris Custer
                    23 hours ago










                  • $begingroup$
                    William Elliot's answer does this.
                    $endgroup$
                    – coffeemath
                    23 hours ago



















                  0












                  $begingroup$

                  Your proof is correct. We can fill in the details on the first part: For any two distinct points $A$ and $B$ in the plane, we can construct a path consisting of unit distances from $A$ to $B$.



                  If $A$ is more than one unit away from $B$, start at $A$ and take unit steps toward $B$ until we are less than one unit from $B$. If $B$ is a whole number of steps away, we will land on $B$. If not, we will land on a point $P$ within the unit circle centered at $B$. Since $P$ is less than a distance of $1$ away from $B$, the unit circle centered at $P$ will intersect the unit circle centered at $B$ in two places. Take one step from $P$ to an intersection point and then one step from the intersection point to $B$ to finish the path.






                  share|cite|improve this answer









                  $endgroup$





















                    0












                    $begingroup$

                    Yes, and here is a very simple and intuitive argument without mirrors ;-)



                    Observation:




                    • Let C be any point on AB such that 0< |AC| < min( |AB|, 2)

                    • We can always travel from A to C in 2 x 1-unit steps. (Construct an appropriate unit side isosceles triangle to a point X on the perpendicular bisector of AC such that |AX| = |XC| = 1).

                    • We can then travel from C to B in < |AB| unit steps.

                    • So all such paths (whatever value of C we chose) will traverse AXCB in < (|AB| + 2) steps, and all such paths are distinct if their respective C' and C are distinct.


                    But as C can be any real number in that range of real numbers, the number of distinct C we could have chosen is uncountable. Each C gives a unique valid path, so the number of paths in uncountable.



                    The followup question would be the cardinality involved. There is a chance it might be more than $mathfrak c=2^{aleph_0}$ [cardinality of reals, corrected per comments below] - but that's far beyond my skill to figure out.






                    share|cite|improve this answer











                    $endgroup$













                    • $begingroup$
                      The cardinality is 'only' aleph_0. You can unique specify any such path by listing the coordinates of all the intermediate points. Hence the set of all paths is contained in the set of finite lists of real numbers. This set has cardinality aleph_0.
                      $endgroup$
                      – quarague
                      21 hours ago










                    • $begingroup$
                      The cardinality of the real numbers is not $aleph_0$. It is $mathfrak c=2^{aleph_0}$. ($aleph_0$ is the cardinality of the integers.)
                      $endgroup$
                      – TonyK
                      18 hours ago












                    • $begingroup$
                      Thanks, both. Especially @quarague - a simple clear answer to that one
                      $endgroup$
                      – Stilez
                      17 hours ago










                    • $begingroup$
                      The correction of TonyK is true. We both meant the cardinality of the reals, but aleph_0 is just the cardinality of the integers.
                      $endgroup$
                      – quarague
                      17 hours ago










                    • $begingroup$
                      Corrected, thank you. But is "set of finite lists" actually the right descriptor? Eg if A=(0,0) B=(0,1) then any specific path of the form [[ ... (-1,0), (-1,0) {n times} ...(0,1), ...(1,0), (1,0) {n times} ... ]] is a valid path. After all, a list of "arbitrary long but finite" paths sounds indistinguishable from an infinite list?
                      $endgroup$
                      – Stilez
                      13 hours ago













                    Your Answer





                    StackExchange.ifUsing("editor", function () {
                    return StackExchange.using("mathjaxEditing", function () {
                    StackExchange.MarkdownEditor.creationCallbacks.add(function (editor, postfix) {
                    StackExchange.mathjaxEditing.prepareWmdForMathJax(editor, postfix, [["$", "$"], ["\\(","\\)"]]);
                    });
                    });
                    }, "mathjax-editing");

                    StackExchange.ready(function() {
                    var channelOptions = {
                    tags: "".split(" "),
                    id: "69"
                    };
                    initTagRenderer("".split(" "), "".split(" "), channelOptions);

                    StackExchange.using("externalEditor", function() {
                    // Have to fire editor after snippets, if snippets enabled
                    if (StackExchange.settings.snippets.snippetsEnabled) {
                    StackExchange.using("snippets", function() {
                    createEditor();
                    });
                    }
                    else {
                    createEditor();
                    }
                    });

                    function createEditor() {
                    StackExchange.prepareEditor({
                    heartbeatType: 'answer',
                    autoActivateHeartbeat: false,
                    convertImagesToLinks: true,
                    noModals: true,
                    showLowRepImageUploadWarning: true,
                    reputationToPostImages: 10,
                    bindNavPrevention: true,
                    postfix: "",
                    imageUploader: {
                    brandingHtml: "Powered by u003ca class="icon-imgur-white" href="https://imgur.com/"u003eu003c/au003e",
                    contentPolicyHtml: "User contributions licensed under u003ca href="https://creativecommons.org/licenses/by-sa/3.0/"u003ecc by-sa 3.0 with attribution requiredu003c/au003e u003ca href="https://stackoverflow.com/legal/content-policy"u003e(content policy)u003c/au003e",
                    allowUrls: true
                    },
                    noCode: true, onDemand: true,
                    discardSelector: ".discard-answer"
                    ,immediatelyShowMarkdownHelp:true
                    });


                    }
                    });














                    draft saved

                    draft discarded


















                    StackExchange.ready(
                    function () {
                    StackExchange.openid.initPostLogin('.new-post-login', 'https%3a%2f%2fmath.stackexchange.com%2fquestions%2f3135697%2fis-the-set-of-paths-between-any-two-points-moving-only-in-units-on-the-plane-cou%23new-answer', 'question_page');
                    }
                    );

                    Post as a guest















                    Required, but never shown

























                    7 Answers
                    7






                    active

                    oldest

                    votes








                    7 Answers
                    7






                    active

                    oldest

                    votes









                    active

                    oldest

                    votes






                    active

                    oldest

                    votes









                    10












                    $begingroup$

                    Yes, this looks convincing. For the missing step, go directly from A towards P in unit steps until the distance left is less than 2. Then use the remaining distance as the base of an isosceles triangle with unit legs, which you make point away from L.






                    share|cite|improve this answer









                    $endgroup$


















                      10












                      $begingroup$

                      Yes, this looks convincing. For the missing step, go directly from A towards P in unit steps until the distance left is less than 2. Then use the remaining distance as the base of an isosceles triangle with unit legs, which you make point away from L.






                      share|cite|improve this answer









                      $endgroup$
















                        10












                        10








                        10





                        $begingroup$

                        Yes, this looks convincing. For the missing step, go directly from A towards P in unit steps until the distance left is less than 2. Then use the remaining distance as the base of an isosceles triangle with unit legs, which you make point away from L.






                        share|cite|improve this answer









                        $endgroup$



                        Yes, this looks convincing. For the missing step, go directly from A towards P in unit steps until the distance left is less than 2. Then use the remaining distance as the base of an isosceles triangle with unit legs, which you make point away from L.







                        share|cite|improve this answer












                        share|cite|improve this answer



                        share|cite|improve this answer










                        answered yesterday









                        Henning MakholmHenning Makholm

                        241k17308548




                        241k17308548























                            5












                            $begingroup$

                            Go one unit from $a$ at an angle of $t$ to $c$.

                            Go in unit steps along ca until one is less than a unit away from $b$ to a point $p$.

                            If $p neq b$, then draw a triangle with base $pb$ and sides of unit length adding the sides as the final steps.



                            As for each $t$ in $[0,2pi)$, I've constructed a different accepted zigzaging from $a$ to $b$, there are uncountably many ways of so staggering from $a$ to $b$.






                            share|cite|improve this answer











                            $endgroup$


















                              5












                              $begingroup$

                              Go one unit from $a$ at an angle of $t$ to $c$.

                              Go in unit steps along ca until one is less than a unit away from $b$ to a point $p$.

                              If $p neq b$, then draw a triangle with base $pb$ and sides of unit length adding the sides as the final steps.



                              As for each $t$ in $[0,2pi)$, I've constructed a different accepted zigzaging from $a$ to $b$, there are uncountably many ways of so staggering from $a$ to $b$.






                              share|cite|improve this answer











                              $endgroup$
















                                5












                                5








                                5





                                $begingroup$

                                Go one unit from $a$ at an angle of $t$ to $c$.

                                Go in unit steps along ca until one is less than a unit away from $b$ to a point $p$.

                                If $p neq b$, then draw a triangle with base $pb$ and sides of unit length adding the sides as the final steps.



                                As for each $t$ in $[0,2pi)$, I've constructed a different accepted zigzaging from $a$ to $b$, there are uncountably many ways of so staggering from $a$ to $b$.






                                share|cite|improve this answer











                                $endgroup$



                                Go one unit from $a$ at an angle of $t$ to $c$.

                                Go in unit steps along ca until one is less than a unit away from $b$ to a point $p$.

                                If $p neq b$, then draw a triangle with base $pb$ and sides of unit length adding the sides as the final steps.



                                As for each $t$ in $[0,2pi)$, I've constructed a different accepted zigzaging from $a$ to $b$, there are uncountably many ways of so staggering from $a$ to $b$.







                                share|cite|improve this answer














                                share|cite|improve this answer



                                share|cite|improve this answer








                                edited 7 hours ago









                                Ed Cottrell

                                158117




                                158117










                                answered yesterday









                                William ElliotWilliam Elliot

                                8,4572720




                                8,4572720























                                    2












                                    $begingroup$

                                    Let $C$ be on the line through $B$ that is perpendicular to the segment $AB$ with the distance $BC$ equal to $1/2.$ Take any half-line $L$, not through $B$, that originates at $A$ and intersects the segment $BC.$ For some $nin Bbb N$ there is a path along $L,$ starting at $A,$ determined by $n$ points $A=A_1,...,A_n$ where $A_j,A_{j+1}$ are distance $1$ apart for each $j<n,$ and such that the distance from $A_n$ to $B$ is less than $1.$



                                    Let the point $D$ be such that $A_nD=BD=1$ nd $Dnot in {A_1,...,A_n}.$ Then the path determined by ${A_1,...,A_n}cup {D}$ is a path of the desired type.



                                    The cardinal of the set all such $L$ is $2^{aleph_0}$ so there at least this many paths of the desired type, joining pairs of points .



                                    And each path is determined by a function from some ${1,2,...,m}subset Bbb N$ into $Bbb R^2.$ The set of all such functions has cardinal $2^{aleph_0}$ so there are at most $2^{aleph_0}$ paths of the desired type.






                                    share|cite|improve this answer











                                    $endgroup$


















                                      2












                                      $begingroup$

                                      Let $C$ be on the line through $B$ that is perpendicular to the segment $AB$ with the distance $BC$ equal to $1/2.$ Take any half-line $L$, not through $B$, that originates at $A$ and intersects the segment $BC.$ For some $nin Bbb N$ there is a path along $L,$ starting at $A,$ determined by $n$ points $A=A_1,...,A_n$ where $A_j,A_{j+1}$ are distance $1$ apart for each $j<n,$ and such that the distance from $A_n$ to $B$ is less than $1.$



                                      Let the point $D$ be such that $A_nD=BD=1$ nd $Dnot in {A_1,...,A_n}.$ Then the path determined by ${A_1,...,A_n}cup {D}$ is a path of the desired type.



                                      The cardinal of the set all such $L$ is $2^{aleph_0}$ so there at least this many paths of the desired type, joining pairs of points .



                                      And each path is determined by a function from some ${1,2,...,m}subset Bbb N$ into $Bbb R^2.$ The set of all such functions has cardinal $2^{aleph_0}$ so there are at most $2^{aleph_0}$ paths of the desired type.






                                      share|cite|improve this answer











                                      $endgroup$
















                                        2












                                        2








                                        2





                                        $begingroup$

                                        Let $C$ be on the line through $B$ that is perpendicular to the segment $AB$ with the distance $BC$ equal to $1/2.$ Take any half-line $L$, not through $B$, that originates at $A$ and intersects the segment $BC.$ For some $nin Bbb N$ there is a path along $L,$ starting at $A,$ determined by $n$ points $A=A_1,...,A_n$ where $A_j,A_{j+1}$ are distance $1$ apart for each $j<n,$ and such that the distance from $A_n$ to $B$ is less than $1.$



                                        Let the point $D$ be such that $A_nD=BD=1$ nd $Dnot in {A_1,...,A_n}.$ Then the path determined by ${A_1,...,A_n}cup {D}$ is a path of the desired type.



                                        The cardinal of the set all such $L$ is $2^{aleph_0}$ so there at least this many paths of the desired type, joining pairs of points .



                                        And each path is determined by a function from some ${1,2,...,m}subset Bbb N$ into $Bbb R^2.$ The set of all such functions has cardinal $2^{aleph_0}$ so there are at most $2^{aleph_0}$ paths of the desired type.






                                        share|cite|improve this answer











                                        $endgroup$



                                        Let $C$ be on the line through $B$ that is perpendicular to the segment $AB$ with the distance $BC$ equal to $1/2.$ Take any half-line $L$, not through $B$, that originates at $A$ and intersects the segment $BC.$ For some $nin Bbb N$ there is a path along $L,$ starting at $A,$ determined by $n$ points $A=A_1,...,A_n$ where $A_j,A_{j+1}$ are distance $1$ apart for each $j<n,$ and such that the distance from $A_n$ to $B$ is less than $1.$



                                        Let the point $D$ be such that $A_nD=BD=1$ nd $Dnot in {A_1,...,A_n}.$ Then the path determined by ${A_1,...,A_n}cup {D}$ is a path of the desired type.



                                        The cardinal of the set all such $L$ is $2^{aleph_0}$ so there at least this many paths of the desired type, joining pairs of points .



                                        And each path is determined by a function from some ${1,2,...,m}subset Bbb N$ into $Bbb R^2.$ The set of all such functions has cardinal $2^{aleph_0}$ so there are at most $2^{aleph_0}$ paths of the desired type.







                                        share|cite|improve this answer














                                        share|cite|improve this answer



                                        share|cite|improve this answer








                                        edited yesterday

























                                        answered yesterday









                                        DanielWainfleetDanielWainfleet

                                        35.3k31648




                                        35.3k31648























                                            1












                                            $begingroup$

                                            Suppose first that the distance $AB$ between $A$ and $B$ is strictly between $2$ and $3$. Then for every point $C$ such that $AC=1$ and $BC<2$, there is a second point $D$ such that $CD=1=BC$ (draw an isosceles triangle). The set of such points $C$ forms an open arc on the unit circle centered at $A$, and so there are uncountably many such $C$, hence uncountably many such "unit paths".



                                            For general points $A$ and $B$, just find one unit path between $A$ and $B$ that contains two intermediate points $A'$ and $B'$ with $2<A'B'<3$ (for example, points on a giant circle eventually connecting $A$ to $B$), and then vary the path from $A'$ to $B'$ in the way described above.



                                            This proof even shows that one can get uncountably many unit paths with the same number of unit-steps. A slight sharpening shows that for any integer $k>AB$, there are uncountably many unit paths from $A$ to $B$ with exactly $k$ unit-steps.






                                            share|cite|improve this answer









                                            $endgroup$


















                                              1












                                              $begingroup$

                                              Suppose first that the distance $AB$ between $A$ and $B$ is strictly between $2$ and $3$. Then for every point $C$ such that $AC=1$ and $BC<2$, there is a second point $D$ such that $CD=1=BC$ (draw an isosceles triangle). The set of such points $C$ forms an open arc on the unit circle centered at $A$, and so there are uncountably many such $C$, hence uncountably many such "unit paths".



                                              For general points $A$ and $B$, just find one unit path between $A$ and $B$ that contains two intermediate points $A'$ and $B'$ with $2<A'B'<3$ (for example, points on a giant circle eventually connecting $A$ to $B$), and then vary the path from $A'$ to $B'$ in the way described above.



                                              This proof even shows that one can get uncountably many unit paths with the same number of unit-steps. A slight sharpening shows that for any integer $k>AB$, there are uncountably many unit paths from $A$ to $B$ with exactly $k$ unit-steps.






                                              share|cite|improve this answer









                                              $endgroup$
















                                                1












                                                1








                                                1





                                                $begingroup$

                                                Suppose first that the distance $AB$ between $A$ and $B$ is strictly between $2$ and $3$. Then for every point $C$ such that $AC=1$ and $BC<2$, there is a second point $D$ such that $CD=1=BC$ (draw an isosceles triangle). The set of such points $C$ forms an open arc on the unit circle centered at $A$, and so there are uncountably many such $C$, hence uncountably many such "unit paths".



                                                For general points $A$ and $B$, just find one unit path between $A$ and $B$ that contains two intermediate points $A'$ and $B'$ with $2<A'B'<3$ (for example, points on a giant circle eventually connecting $A$ to $B$), and then vary the path from $A'$ to $B'$ in the way described above.



                                                This proof even shows that one can get uncountably many unit paths with the same number of unit-steps. A slight sharpening shows that for any integer $k>AB$, there are uncountably many unit paths from $A$ to $B$ with exactly $k$ unit-steps.






                                                share|cite|improve this answer









                                                $endgroup$



                                                Suppose first that the distance $AB$ between $A$ and $B$ is strictly between $2$ and $3$. Then for every point $C$ such that $AC=1$ and $BC<2$, there is a second point $D$ such that $CD=1=BC$ (draw an isosceles triangle). The set of such points $C$ forms an open arc on the unit circle centered at $A$, and so there are uncountably many such $C$, hence uncountably many such "unit paths".



                                                For general points $A$ and $B$, just find one unit path between $A$ and $B$ that contains two intermediate points $A'$ and $B'$ with $2<A'B'<3$ (for example, points on a giant circle eventually connecting $A$ to $B$), and then vary the path from $A'$ to $B'$ in the way described above.



                                                This proof even shows that one can get uncountably many unit paths with the same number of unit-steps. A slight sharpening shows that for any integer $k>AB$, there are uncountably many unit paths from $A$ to $B$ with exactly $k$ unit-steps.







                                                share|cite|improve this answer












                                                share|cite|improve this answer



                                                share|cite|improve this answer










                                                answered 19 hours ago









                                                Greg MartinGreg Martin

                                                36k23565




                                                36k23565























                                                    0












                                                    $begingroup$

                                                    I believe it is uncountable also, and here is my thought process:



                                                    For each direction vector $vinBbb R^2=T_aBbb R^2$, take a curve $alpha_v$ from $a$ to $b$ with $alpha_v'(0)=v$.



                                                    Then if $vneq w$, we have $alpha_vneqalpha_w$.



                                                    But clearly there are uncountably many direction vectors in $Bbb R^2=T_aBbb R^2$.






                                                    share|cite|improve this answer









                                                    $endgroup$













                                                    • $begingroup$
                                                      It seems OP wants movements made of straight line segments all of length $1,$ to go from fixed $A$ to fixed $b$ points. [rather than curves as you use]
                                                      $endgroup$
                                                      – coffeemath
                                                      yesterday










                                                    • $begingroup$
                                                      I did notice that. But I thought, well, straight lines have derivatives. @coffeemath
                                                      $endgroup$
                                                      – Chris Custer
                                                      yesterday










                                                    • $begingroup$
                                                      Cris-- Then to finish one has to show that after the first initial unit straight line step from $A,$ it is possible to continue such steps and somehow arrive at $B.$
                                                      $endgroup$
                                                      – coffeemath
                                                      23 hours ago










                                                    • $begingroup$
                                                      @coffeemath yes. Well there certainly appear to be some details missing. But this seems fairly reasonable. Thanks.
                                                      $endgroup$
                                                      – Chris Custer
                                                      23 hours ago










                                                    • $begingroup$
                                                      William Elliot's answer does this.
                                                      $endgroup$
                                                      – coffeemath
                                                      23 hours ago
















                                                    0












                                                    $begingroup$

                                                    I believe it is uncountable also, and here is my thought process:



                                                    For each direction vector $vinBbb R^2=T_aBbb R^2$, take a curve $alpha_v$ from $a$ to $b$ with $alpha_v'(0)=v$.



                                                    Then if $vneq w$, we have $alpha_vneqalpha_w$.



                                                    But clearly there are uncountably many direction vectors in $Bbb R^2=T_aBbb R^2$.






                                                    share|cite|improve this answer









                                                    $endgroup$













                                                    • $begingroup$
                                                      It seems OP wants movements made of straight line segments all of length $1,$ to go from fixed $A$ to fixed $b$ points. [rather than curves as you use]
                                                      $endgroup$
                                                      – coffeemath
                                                      yesterday










                                                    • $begingroup$
                                                      I did notice that. But I thought, well, straight lines have derivatives. @coffeemath
                                                      $endgroup$
                                                      – Chris Custer
                                                      yesterday










                                                    • $begingroup$
                                                      Cris-- Then to finish one has to show that after the first initial unit straight line step from $A,$ it is possible to continue such steps and somehow arrive at $B.$
                                                      $endgroup$
                                                      – coffeemath
                                                      23 hours ago










                                                    • $begingroup$
                                                      @coffeemath yes. Well there certainly appear to be some details missing. But this seems fairly reasonable. Thanks.
                                                      $endgroup$
                                                      – Chris Custer
                                                      23 hours ago










                                                    • $begingroup$
                                                      William Elliot's answer does this.
                                                      $endgroup$
                                                      – coffeemath
                                                      23 hours ago














                                                    0












                                                    0








                                                    0





                                                    $begingroup$

                                                    I believe it is uncountable also, and here is my thought process:



                                                    For each direction vector $vinBbb R^2=T_aBbb R^2$, take a curve $alpha_v$ from $a$ to $b$ with $alpha_v'(0)=v$.



                                                    Then if $vneq w$, we have $alpha_vneqalpha_w$.



                                                    But clearly there are uncountably many direction vectors in $Bbb R^2=T_aBbb R^2$.






                                                    share|cite|improve this answer









                                                    $endgroup$



                                                    I believe it is uncountable also, and here is my thought process:



                                                    For each direction vector $vinBbb R^2=T_aBbb R^2$, take a curve $alpha_v$ from $a$ to $b$ with $alpha_v'(0)=v$.



                                                    Then if $vneq w$, we have $alpha_vneqalpha_w$.



                                                    But clearly there are uncountably many direction vectors in $Bbb R^2=T_aBbb R^2$.







                                                    share|cite|improve this answer












                                                    share|cite|improve this answer



                                                    share|cite|improve this answer










                                                    answered yesterday









                                                    Chris CusterChris Custer

                                                    14k3827




                                                    14k3827












                                                    • $begingroup$
                                                      It seems OP wants movements made of straight line segments all of length $1,$ to go from fixed $A$ to fixed $b$ points. [rather than curves as you use]
                                                      $endgroup$
                                                      – coffeemath
                                                      yesterday










                                                    • $begingroup$
                                                      I did notice that. But I thought, well, straight lines have derivatives. @coffeemath
                                                      $endgroup$
                                                      – Chris Custer
                                                      yesterday










                                                    • $begingroup$
                                                      Cris-- Then to finish one has to show that after the first initial unit straight line step from $A,$ it is possible to continue such steps and somehow arrive at $B.$
                                                      $endgroup$
                                                      – coffeemath
                                                      23 hours ago










                                                    • $begingroup$
                                                      @coffeemath yes. Well there certainly appear to be some details missing. But this seems fairly reasonable. Thanks.
                                                      $endgroup$
                                                      – Chris Custer
                                                      23 hours ago










                                                    • $begingroup$
                                                      William Elliot's answer does this.
                                                      $endgroup$
                                                      – coffeemath
                                                      23 hours ago


















                                                    • $begingroup$
                                                      It seems OP wants movements made of straight line segments all of length $1,$ to go from fixed $A$ to fixed $b$ points. [rather than curves as you use]
                                                      $endgroup$
                                                      – coffeemath
                                                      yesterday










                                                    • $begingroup$
                                                      I did notice that. But I thought, well, straight lines have derivatives. @coffeemath
                                                      $endgroup$
                                                      – Chris Custer
                                                      yesterday










                                                    • $begingroup$
                                                      Cris-- Then to finish one has to show that after the first initial unit straight line step from $A,$ it is possible to continue such steps and somehow arrive at $B.$
                                                      $endgroup$
                                                      – coffeemath
                                                      23 hours ago










                                                    • $begingroup$
                                                      @coffeemath yes. Well there certainly appear to be some details missing. But this seems fairly reasonable. Thanks.
                                                      $endgroup$
                                                      – Chris Custer
                                                      23 hours ago










                                                    • $begingroup$
                                                      William Elliot's answer does this.
                                                      $endgroup$
                                                      – coffeemath
                                                      23 hours ago
















                                                    $begingroup$
                                                    It seems OP wants movements made of straight line segments all of length $1,$ to go from fixed $A$ to fixed $b$ points. [rather than curves as you use]
                                                    $endgroup$
                                                    – coffeemath
                                                    yesterday




                                                    $begingroup$
                                                    It seems OP wants movements made of straight line segments all of length $1,$ to go from fixed $A$ to fixed $b$ points. [rather than curves as you use]
                                                    $endgroup$
                                                    – coffeemath
                                                    yesterday












                                                    $begingroup$
                                                    I did notice that. But I thought, well, straight lines have derivatives. @coffeemath
                                                    $endgroup$
                                                    – Chris Custer
                                                    yesterday




                                                    $begingroup$
                                                    I did notice that. But I thought, well, straight lines have derivatives. @coffeemath
                                                    $endgroup$
                                                    – Chris Custer
                                                    yesterday












                                                    $begingroup$
                                                    Cris-- Then to finish one has to show that after the first initial unit straight line step from $A,$ it is possible to continue such steps and somehow arrive at $B.$
                                                    $endgroup$
                                                    – coffeemath
                                                    23 hours ago




                                                    $begingroup$
                                                    Cris-- Then to finish one has to show that after the first initial unit straight line step from $A,$ it is possible to continue such steps and somehow arrive at $B.$
                                                    $endgroup$
                                                    – coffeemath
                                                    23 hours ago












                                                    $begingroup$
                                                    @coffeemath yes. Well there certainly appear to be some details missing. But this seems fairly reasonable. Thanks.
                                                    $endgroup$
                                                    – Chris Custer
                                                    23 hours ago




                                                    $begingroup$
                                                    @coffeemath yes. Well there certainly appear to be some details missing. But this seems fairly reasonable. Thanks.
                                                    $endgroup$
                                                    – Chris Custer
                                                    23 hours ago












                                                    $begingroup$
                                                    William Elliot's answer does this.
                                                    $endgroup$
                                                    – coffeemath
                                                    23 hours ago




                                                    $begingroup$
                                                    William Elliot's answer does this.
                                                    $endgroup$
                                                    – coffeemath
                                                    23 hours ago











                                                    0












                                                    $begingroup$

                                                    Your proof is correct. We can fill in the details on the first part: For any two distinct points $A$ and $B$ in the plane, we can construct a path consisting of unit distances from $A$ to $B$.



                                                    If $A$ is more than one unit away from $B$, start at $A$ and take unit steps toward $B$ until we are less than one unit from $B$. If $B$ is a whole number of steps away, we will land on $B$. If not, we will land on a point $P$ within the unit circle centered at $B$. Since $P$ is less than a distance of $1$ away from $B$, the unit circle centered at $P$ will intersect the unit circle centered at $B$ in two places. Take one step from $P$ to an intersection point and then one step from the intersection point to $B$ to finish the path.






                                                    share|cite|improve this answer









                                                    $endgroup$


















                                                      0












                                                      $begingroup$

                                                      Your proof is correct. We can fill in the details on the first part: For any two distinct points $A$ and $B$ in the plane, we can construct a path consisting of unit distances from $A$ to $B$.



                                                      If $A$ is more than one unit away from $B$, start at $A$ and take unit steps toward $B$ until we are less than one unit from $B$. If $B$ is a whole number of steps away, we will land on $B$. If not, we will land on a point $P$ within the unit circle centered at $B$. Since $P$ is less than a distance of $1$ away from $B$, the unit circle centered at $P$ will intersect the unit circle centered at $B$ in two places. Take one step from $P$ to an intersection point and then one step from the intersection point to $B$ to finish the path.






                                                      share|cite|improve this answer









                                                      $endgroup$
















                                                        0












                                                        0








                                                        0





                                                        $begingroup$

                                                        Your proof is correct. We can fill in the details on the first part: For any two distinct points $A$ and $B$ in the plane, we can construct a path consisting of unit distances from $A$ to $B$.



                                                        If $A$ is more than one unit away from $B$, start at $A$ and take unit steps toward $B$ until we are less than one unit from $B$. If $B$ is a whole number of steps away, we will land on $B$. If not, we will land on a point $P$ within the unit circle centered at $B$. Since $P$ is less than a distance of $1$ away from $B$, the unit circle centered at $P$ will intersect the unit circle centered at $B$ in two places. Take one step from $P$ to an intersection point and then one step from the intersection point to $B$ to finish the path.






                                                        share|cite|improve this answer









                                                        $endgroup$



                                                        Your proof is correct. We can fill in the details on the first part: For any two distinct points $A$ and $B$ in the plane, we can construct a path consisting of unit distances from $A$ to $B$.



                                                        If $A$ is more than one unit away from $B$, start at $A$ and take unit steps toward $B$ until we are less than one unit from $B$. If $B$ is a whole number of steps away, we will land on $B$. If not, we will land on a point $P$ within the unit circle centered at $B$. Since $P$ is less than a distance of $1$ away from $B$, the unit circle centered at $P$ will intersect the unit circle centered at $B$ in two places. Take one step from $P$ to an intersection point and then one step from the intersection point to $B$ to finish the path.







                                                        share|cite|improve this answer












                                                        share|cite|improve this answer



                                                        share|cite|improve this answer










                                                        answered yesterday









                                                        John DoumaJohn Douma

                                                        5,57211319




                                                        5,57211319























                                                            0












                                                            $begingroup$

                                                            Yes, and here is a very simple and intuitive argument without mirrors ;-)



                                                            Observation:




                                                            • Let C be any point on AB such that 0< |AC| < min( |AB|, 2)

                                                            • We can always travel from A to C in 2 x 1-unit steps. (Construct an appropriate unit side isosceles triangle to a point X on the perpendicular bisector of AC such that |AX| = |XC| = 1).

                                                            • We can then travel from C to B in < |AB| unit steps.

                                                            • So all such paths (whatever value of C we chose) will traverse AXCB in < (|AB| + 2) steps, and all such paths are distinct if their respective C' and C are distinct.


                                                            But as C can be any real number in that range of real numbers, the number of distinct C we could have chosen is uncountable. Each C gives a unique valid path, so the number of paths in uncountable.



                                                            The followup question would be the cardinality involved. There is a chance it might be more than $mathfrak c=2^{aleph_0}$ [cardinality of reals, corrected per comments below] - but that's far beyond my skill to figure out.






                                                            share|cite|improve this answer











                                                            $endgroup$













                                                            • $begingroup$
                                                              The cardinality is 'only' aleph_0. You can unique specify any such path by listing the coordinates of all the intermediate points. Hence the set of all paths is contained in the set of finite lists of real numbers. This set has cardinality aleph_0.
                                                              $endgroup$
                                                              – quarague
                                                              21 hours ago










                                                            • $begingroup$
                                                              The cardinality of the real numbers is not $aleph_0$. It is $mathfrak c=2^{aleph_0}$. ($aleph_0$ is the cardinality of the integers.)
                                                              $endgroup$
                                                              – TonyK
                                                              18 hours ago












                                                            • $begingroup$
                                                              Thanks, both. Especially @quarague - a simple clear answer to that one
                                                              $endgroup$
                                                              – Stilez
                                                              17 hours ago










                                                            • $begingroup$
                                                              The correction of TonyK is true. We both meant the cardinality of the reals, but aleph_0 is just the cardinality of the integers.
                                                              $endgroup$
                                                              – quarague
                                                              17 hours ago










                                                            • $begingroup$
                                                              Corrected, thank you. But is "set of finite lists" actually the right descriptor? Eg if A=(0,0) B=(0,1) then any specific path of the form [[ ... (-1,0), (-1,0) {n times} ...(0,1), ...(1,0), (1,0) {n times} ... ]] is a valid path. After all, a list of "arbitrary long but finite" paths sounds indistinguishable from an infinite list?
                                                              $endgroup$
                                                              – Stilez
                                                              13 hours ago


















                                                            0












                                                            $begingroup$

                                                            Yes, and here is a very simple and intuitive argument without mirrors ;-)



                                                            Observation:




                                                            • Let C be any point on AB such that 0< |AC| < min( |AB|, 2)

                                                            • We can always travel from A to C in 2 x 1-unit steps. (Construct an appropriate unit side isosceles triangle to a point X on the perpendicular bisector of AC such that |AX| = |XC| = 1).

                                                            • We can then travel from C to B in < |AB| unit steps.

                                                            • So all such paths (whatever value of C we chose) will traverse AXCB in < (|AB| + 2) steps, and all such paths are distinct if their respective C' and C are distinct.


                                                            But as C can be any real number in that range of real numbers, the number of distinct C we could have chosen is uncountable. Each C gives a unique valid path, so the number of paths in uncountable.



                                                            The followup question would be the cardinality involved. There is a chance it might be more than $mathfrak c=2^{aleph_0}$ [cardinality of reals, corrected per comments below] - but that's far beyond my skill to figure out.






                                                            share|cite|improve this answer











                                                            $endgroup$













                                                            • $begingroup$
                                                              The cardinality is 'only' aleph_0. You can unique specify any such path by listing the coordinates of all the intermediate points. Hence the set of all paths is contained in the set of finite lists of real numbers. This set has cardinality aleph_0.
                                                              $endgroup$
                                                              – quarague
                                                              21 hours ago










                                                            • $begingroup$
                                                              The cardinality of the real numbers is not $aleph_0$. It is $mathfrak c=2^{aleph_0}$. ($aleph_0$ is the cardinality of the integers.)
                                                              $endgroup$
                                                              – TonyK
                                                              18 hours ago












                                                            • $begingroup$
                                                              Thanks, both. Especially @quarague - a simple clear answer to that one
                                                              $endgroup$
                                                              – Stilez
                                                              17 hours ago










                                                            • $begingroup$
                                                              The correction of TonyK is true. We both meant the cardinality of the reals, but aleph_0 is just the cardinality of the integers.
                                                              $endgroup$
                                                              – quarague
                                                              17 hours ago










                                                            • $begingroup$
                                                              Corrected, thank you. But is "set of finite lists" actually the right descriptor? Eg if A=(0,0) B=(0,1) then any specific path of the form [[ ... (-1,0), (-1,0) {n times} ...(0,1), ...(1,0), (1,0) {n times} ... ]] is a valid path. After all, a list of "arbitrary long but finite" paths sounds indistinguishable from an infinite list?
                                                              $endgroup$
                                                              – Stilez
                                                              13 hours ago
















                                                            0












                                                            0








                                                            0





                                                            $begingroup$

                                                            Yes, and here is a very simple and intuitive argument without mirrors ;-)



                                                            Observation:




                                                            • Let C be any point on AB such that 0< |AC| < min( |AB|, 2)

                                                            • We can always travel from A to C in 2 x 1-unit steps. (Construct an appropriate unit side isosceles triangle to a point X on the perpendicular bisector of AC such that |AX| = |XC| = 1).

                                                            • We can then travel from C to B in < |AB| unit steps.

                                                            • So all such paths (whatever value of C we chose) will traverse AXCB in < (|AB| + 2) steps, and all such paths are distinct if their respective C' and C are distinct.


                                                            But as C can be any real number in that range of real numbers, the number of distinct C we could have chosen is uncountable. Each C gives a unique valid path, so the number of paths in uncountable.



                                                            The followup question would be the cardinality involved. There is a chance it might be more than $mathfrak c=2^{aleph_0}$ [cardinality of reals, corrected per comments below] - but that's far beyond my skill to figure out.






                                                            share|cite|improve this answer











                                                            $endgroup$



                                                            Yes, and here is a very simple and intuitive argument without mirrors ;-)



                                                            Observation:




                                                            • Let C be any point on AB such that 0< |AC| < min( |AB|, 2)

                                                            • We can always travel from A to C in 2 x 1-unit steps. (Construct an appropriate unit side isosceles triangle to a point X on the perpendicular bisector of AC such that |AX| = |XC| = 1).

                                                            • We can then travel from C to B in < |AB| unit steps.

                                                            • So all such paths (whatever value of C we chose) will traverse AXCB in < (|AB| + 2) steps, and all such paths are distinct if their respective C' and C are distinct.


                                                            But as C can be any real number in that range of real numbers, the number of distinct C we could have chosen is uncountable. Each C gives a unique valid path, so the number of paths in uncountable.



                                                            The followup question would be the cardinality involved. There is a chance it might be more than $mathfrak c=2^{aleph_0}$ [cardinality of reals, corrected per comments below] - but that's far beyond my skill to figure out.







                                                            share|cite|improve this answer














                                                            share|cite|improve this answer



                                                            share|cite|improve this answer








                                                            edited 13 hours ago

























                                                            answered 22 hours ago









                                                            StilezStilez

                                                            42129




                                                            42129












                                                            • $begingroup$
                                                              The cardinality is 'only' aleph_0. You can unique specify any such path by listing the coordinates of all the intermediate points. Hence the set of all paths is contained in the set of finite lists of real numbers. This set has cardinality aleph_0.
                                                              $endgroup$
                                                              – quarague
                                                              21 hours ago










                                                            • $begingroup$
                                                              The cardinality of the real numbers is not $aleph_0$. It is $mathfrak c=2^{aleph_0}$. ($aleph_0$ is the cardinality of the integers.)
                                                              $endgroup$
                                                              – TonyK
                                                              18 hours ago












                                                            • $begingroup$
                                                              Thanks, both. Especially @quarague - a simple clear answer to that one
                                                              $endgroup$
                                                              – Stilez
                                                              17 hours ago










                                                            • $begingroup$
                                                              The correction of TonyK is true. We both meant the cardinality of the reals, but aleph_0 is just the cardinality of the integers.
                                                              $endgroup$
                                                              – quarague
                                                              17 hours ago










                                                            • $begingroup$
                                                              Corrected, thank you. But is "set of finite lists" actually the right descriptor? Eg if A=(0,0) B=(0,1) then any specific path of the form [[ ... (-1,0), (-1,0) {n times} ...(0,1), ...(1,0), (1,0) {n times} ... ]] is a valid path. After all, a list of "arbitrary long but finite" paths sounds indistinguishable from an infinite list?
                                                              $endgroup$
                                                              – Stilez
                                                              13 hours ago




















                                                            • $begingroup$
                                                              The cardinality is 'only' aleph_0. You can unique specify any such path by listing the coordinates of all the intermediate points. Hence the set of all paths is contained in the set of finite lists of real numbers. This set has cardinality aleph_0.
                                                              $endgroup$
                                                              – quarague
                                                              21 hours ago










                                                            • $begingroup$
                                                              The cardinality of the real numbers is not $aleph_0$. It is $mathfrak c=2^{aleph_0}$. ($aleph_0$ is the cardinality of the integers.)
                                                              $endgroup$
                                                              – TonyK
                                                              18 hours ago












                                                            • $begingroup$
                                                              Thanks, both. Especially @quarague - a simple clear answer to that one
                                                              $endgroup$
                                                              – Stilez
                                                              17 hours ago










                                                            • $begingroup$
                                                              The correction of TonyK is true. We both meant the cardinality of the reals, but aleph_0 is just the cardinality of the integers.
                                                              $endgroup$
                                                              – quarague
                                                              17 hours ago










                                                            • $begingroup$
                                                              Corrected, thank you. But is "set of finite lists" actually the right descriptor? Eg if A=(0,0) B=(0,1) then any specific path of the form [[ ... (-1,0), (-1,0) {n times} ...(0,1), ...(1,0), (1,0) {n times} ... ]] is a valid path. After all, a list of "arbitrary long but finite" paths sounds indistinguishable from an infinite list?
                                                              $endgroup$
                                                              – Stilez
                                                              13 hours ago


















                                                            $begingroup$
                                                            The cardinality is 'only' aleph_0. You can unique specify any such path by listing the coordinates of all the intermediate points. Hence the set of all paths is contained in the set of finite lists of real numbers. This set has cardinality aleph_0.
                                                            $endgroup$
                                                            – quarague
                                                            21 hours ago




                                                            $begingroup$
                                                            The cardinality is 'only' aleph_0. You can unique specify any such path by listing the coordinates of all the intermediate points. Hence the set of all paths is contained in the set of finite lists of real numbers. This set has cardinality aleph_0.
                                                            $endgroup$
                                                            – quarague
                                                            21 hours ago












                                                            $begingroup$
                                                            The cardinality of the real numbers is not $aleph_0$. It is $mathfrak c=2^{aleph_0}$. ($aleph_0$ is the cardinality of the integers.)
                                                            $endgroup$
                                                            – TonyK
                                                            18 hours ago






                                                            $begingroup$
                                                            The cardinality of the real numbers is not $aleph_0$. It is $mathfrak c=2^{aleph_0}$. ($aleph_0$ is the cardinality of the integers.)
                                                            $endgroup$
                                                            – TonyK
                                                            18 hours ago














                                                            $begingroup$
                                                            Thanks, both. Especially @quarague - a simple clear answer to that one
                                                            $endgroup$
                                                            – Stilez
                                                            17 hours ago




                                                            $begingroup$
                                                            Thanks, both. Especially @quarague - a simple clear answer to that one
                                                            $endgroup$
                                                            – Stilez
                                                            17 hours ago












                                                            $begingroup$
                                                            The correction of TonyK is true. We both meant the cardinality of the reals, but aleph_0 is just the cardinality of the integers.
                                                            $endgroup$
                                                            – quarague
                                                            17 hours ago




                                                            $begingroup$
                                                            The correction of TonyK is true. We both meant the cardinality of the reals, but aleph_0 is just the cardinality of the integers.
                                                            $endgroup$
                                                            – quarague
                                                            17 hours ago












                                                            $begingroup$
                                                            Corrected, thank you. But is "set of finite lists" actually the right descriptor? Eg if A=(0,0) B=(0,1) then any specific path of the form [[ ... (-1,0), (-1,0) {n times} ...(0,1), ...(1,0), (1,0) {n times} ... ]] is a valid path. After all, a list of "arbitrary long but finite" paths sounds indistinguishable from an infinite list?
                                                            $endgroup$
                                                            – Stilez
                                                            13 hours ago






                                                            $begingroup$
                                                            Corrected, thank you. But is "set of finite lists" actually the right descriptor? Eg if A=(0,0) B=(0,1) then any specific path of the form [[ ... (-1,0), (-1,0) {n times} ...(0,1), ...(1,0), (1,0) {n times} ... ]] is a valid path. After all, a list of "arbitrary long but finite" paths sounds indistinguishable from an infinite list?
                                                            $endgroup$
                                                            – Stilez
                                                            13 hours ago




















                                                            draft saved

                                                            draft discarded




















































                                                            Thanks for contributing an answer to Mathematics Stack Exchange!


                                                            • Please be sure to answer the question. Provide details and share your research!

                                                            But avoid



                                                            • Asking for help, clarification, or responding to other answers.

                                                            • Making statements based on opinion; back them up with references or personal experience.


                                                            Use MathJax to format equations. MathJax reference.


                                                            To learn more, see our tips on writing great answers.




                                                            draft saved


                                                            draft discarded














                                                            StackExchange.ready(
                                                            function () {
                                                            StackExchange.openid.initPostLogin('.new-post-login', 'https%3a%2f%2fmath.stackexchange.com%2fquestions%2f3135697%2fis-the-set-of-paths-between-any-two-points-moving-only-in-units-on-the-plane-cou%23new-answer', 'question_page');
                                                            }
                                                            );

                                                            Post as a guest















                                                            Required, but never shown





















































                                                            Required, but never shown














                                                            Required, but never shown












                                                            Required, but never shown







                                                            Required, but never shown

































                                                            Required, but never shown














                                                            Required, but never shown












                                                            Required, but never shown







                                                            Required, but never shown







                                                            Popular posts from this blog

                                                            VNC viewer RFB protocol error: bad desktop size 0x0I Cannot Type the Key 'd' (lowercase) in VNC Viewer...

                                                            Tribunal Administrativo e Fiscal de Mirandela Referências Menu de...

                                                            looking for continuous Screen Capture for retroactivly reproducing errors, timeback machineRolling desktop...